LSAT and Law School Admissions Forum

Get expert LSAT preparation and law school admissions advice from PowerScore Test Preparation.

User avatar
 Dave Killoran
PowerScore Staff
  • PowerScore Staff
  • Posts: 5849
  • Joined: Mar 25, 2011
|
#59768
Complete Question Explanation
(The complete setup for this game can be found here: lsat/viewtopic.php?t=1753)

The correct answer choice is (D)

This is a straightforward List question, and a great way to start a difficult game.

Answer choice (A) is incorrect because J cannot be in area R (J is already in area S, and S and R do not overlap according to the game scenario).

Answer choice (B) is incorrect because from the second rule J must be in area S, and because J cannot be in area R (as explained in answer choice (A)).

Answer choice (C) is incorrect because, due to the last rule, M cannot be in area U.

Answer choice (D) is the correct answer.

Answer choice (E) is incorrect because, due to the second and third rules, K cannot be in area S.

Get the most out of your LSAT Prep Plus subscription.

Analyze and track your performance with our Testing and Analytics Package.